Chapter 30 - Pain Assessment and Management in Children

Pataasin ang iyong marka sa homework at exams ngayon gamit ang Quizwiz!

The nurse is preparing to administer an IM injection to a 5 year old and asks the student working with the patient to assist by blowing bubbles for the patient while the nurse gives the injection. Which type of pain management is blowing bubbles? a) relaxation b) thought stopping c) distraction d) imagery

distraction Correct Explanation: Distraction techniques aim at shifting a child's focus from pain to another activity or interest, In this case, blowing bubbles serves as a distraction.

The nurse is caring for a child with appendicitis. The nurse understands that this child is experiencing what type of pain? Select all that apply. a) Acute pain b) Neuropathic pain c) Deep somatic pain d) Visceral pain

• Acute pain • Visceral pain Explanation: Visceral pain is pain that develops within organs such as the heart, lungs, gastrointestinal tract, pancreas, liver, gallbladder, kidneys, or bladder. Acute pain is pain usually associated with rapid onset, trauma, or surgery. It lasts a few days. Deep somatic pain usually involves the muscles, tendons, joints, fascia, and bones. Neuropathic pain is due to malfunctioning of the peripheral or central nervous system

The physician has ordered the postoperative four-year-old child to receive hydromorphine (Dilaudid) intravenously. The drug book lists a therapeutic range for Dilaudid to be 0.01 to 0.015 mg/kg/dose every three to four hours. What would be the maximum therapeutic dose of Dilaudid if the child weighs 30 pounds? Round your answer to the nearest hundredth.

0.2 mg

B. Administer docusate sodium (Colace). C. Encourage fluid intake. D. Encourage the child to eat fruit. Administration of Colace, a stool softener, can help prevent constipation. Increased fluid and fruit intake (high fiber content) can help prevent constipation. Increased activity helps stimulate peristalsis. Diphenhydramine would not increase peristalsis or prevent constipation.

A 6-year-old is hospitalized with a fractured femur. Based on the nurse's knowledge of opioid side effects, the nurse should include which actions in the patient's plan of care to prevent constipation? (Select all that apply.) A. Instruct the child to remain supine while in bed. B. Administer docusate sodium (Colace). C. Encourage fluid intake. D. Encourage the child to eat fruit. E. Administer diphenhydramine (Benadryl).

A five-year-old is hospitalized with a fractured femur. Which assessment tool is appropriate for this child? A. CRIES Scale B. Faces Pain Rating Scale C. SUN Scale D. PIPP Scale

B. Faces Pain Rating Scale A five-year-old child should be able to use the Faces Scale to choose which face best matches the child's pain level. The CRIES Scale was developed for preterm and full-term neonates. The SUN Scale was developed for use in newborns. The PIPP Scale was developed for premature infants.

The 17-month-old infant is terminally ill with cancer and is in constant pain. The nurse recognizes that the best way to control pain in this child would be for the physician to order: A. Patient-controlled analgesia with the parents controlling the button that administers the dosage B. Intravenously administered opioids on a scheduled basis C. Intravenously administered opioids on a prn basis D. Parenteral administration controls pain more effectively than oral medication as oral absorption may be modified by stomach activities. In addition, providing analgesics on a scheduled basis is preferred over prn

B. Intravenously administered opioids on a scheduled basis This provides continuous blood levels of the opioid. PCA should always be controlled by the individual receiving the medication. It is inappropriate to have the parents control the medication administration. By waiting until symptoms are present, the child's blood level will drop, making it more difficult to control the pain.

The nurse is preparing to perform a heel stick on a neonate. The most appropriate complementary therapy for the nurse to plan to use in the neonate to decrease pain during this quick but painful procedure is: A. Holding the infant B. Sucrose pacifier C. Massage D. Swaddling

B. Sucrose pacifier Sucrose provides short-term, natural pain relief, and is most appropriate for use in neonates to decrease pain associated with a quick procedure. Massage and holding the infant are more appropriate following the procedure, or as an adjunct to pain medication for ongoing pain or distress. Swaddling for a neonate undergoing a quick painful procedure will not decrease the pain.

A six-year-old postoperative patient IV has infiltrated and has to be restarted immediately for medication. There is no time for placing local anesthetic cream on the skin. What other complementary therapies would be most helpful when placing this IV? A. Restraints B. Moderate sedation C. Anesthesia D. Distraction

D. Distraction Complementary therapies—especially guided imagery, relaxation techniques, and distraction—can reduce the anxiety associated with the anticipation of the procedure. Restraints are used only as a last resort and are not appropriate for an IV start. Moderate sedation has its own side effects and possible complications and should not be used for quick procedures. Drugs may not be used for quick procedures, such as a dressing change or an unexpected intravenous insertion, injection, or venipuncture.

The nurse is caring for a two-year-old child in the postoperative period. The pain assessment tool most appropriate for assessment of pain intensity in a two-year-old is the: A. Poker Chip Tool B. Oucher Scale C. Faces Pain Rating Scale D. FLACC Behavioral Pain Assessment Scale

D. FLACC Behavioral Pain Assessment Scale The FLACC scale is an appropriate tool for infants and young children who cannot report pain. The Faces Scale, Oucher Scale, and Poker Chip Tool are all self-report scales and can usually be used with children three and older.

The postoperative unit of the pediatric hospital has several children who had surgery this morning. While making rounds, the nurse observes all of the following behaviors. Which child should be further evaluated as to postoperative pain? A. The six-month-old in deep sleep B. The two-year-old who is cooperative when the nurse takes his vital signs C. The four-year-old who is actively watching cartoons D. The 14-month-old who is thrashing his arms and legs

D. The 14-month-old who is thrashing his arms and legs Young children in acute pain display a variety of behaviors, including loud crying, screaming, thrashing their arms and legs, lack of cooperation, clinging behavior, and restlessness and irritability. Children are unable to obtain deep sleep when experiencing acute pain. When experiencing acute pain, children are less likely to cooperate with treatments. It is difficult for children to concentrate when experiencing acute pain.

The nurse is providing postsurgical care for a 4-year-old boy following hernia repair. Before surgery, the nurse taught the child to use the poker chip tool to rate his pain. When assessing the child's postsurgical pain, the boy refuses to touch the chips and clings to his mother. How should the nurse respond? a) Substitute the word-graphic rating scale for the poker chips. b) Give the mother the FACES pain rating scale to use with her son. c) Show the child once more how to use the chips. d) Select the visual analog scale as the best one to use.

Give the mother the FACES pain rating scale to use with her son. Correct Explanation: Different pain rating scales are appropriate for different developmental levels. Children often regress when in pain, so a simpler tool such as the FACES scale may be needed. It is also helpful to enlist the assistance of the parent. Expecting the child to select a chip is developmentally inappropriate when the child shows signs of regression. The child wouldn't understand the phrase "word-graphic scale," and this scale or the visual analog scale is more complex than this 4-year-old can handle

Which method of pain tools can be used to assess the pain in relation to events and times of day? a) Adolescent Pediatric Pain Tool b) FACES Pain Rating Scale c) Log and diaries d) Numerical or Visual Analog Scale

Log and diaries Correct Explanation: The log and diaries is a method used to chart or log when pain occurs. The child or parent can include information about events or locations when the pain occurs along with other items to note. The other options are pain-assessment tools to use with acute-pain events.

The nurse is assessing an adolescent for a rule-out appendicitis. The nurse is aware the appendix is located in the right lower quadrant. The teenager is complaining of pain in the left lower quadrant. What type of pain should the nurse document? a) Referred pain b) Chronic pain c) Localized pain d) Cutaneous pain

Referred pain Correct Explanation: Referred pain is pain that is perceived at a site distant from the point of origin. Appendicitis appears with referred pain during assessment. Chronic pain is ongoing, localized pain is with a superficial injury, and cutaneous pain is another name for localized pain.

A nurse is interviewing the mother of a sleeping 10 year old girl in order to asses the level of the child's post-operative pain. Which of the following comments would trigger additional questions and necessitate further teaching? a. She is asleep, so she must not be in pain b. she has never had surgery before c. she is very articulate and will tell you how she feels d. she has a very easy going termperament

a. she is asleep so she must not be in pain

A nurse is assessing the pain level of an infant. Which of the following findings is not a typical physiologic indicator of pain? a. decreased oxygen saturation b. decreased heart rate c. palmar sweating d. plantar sweating

b. decreased heart rate

The nurse is providing family education for the administration of ibuprofen. Which of the following responses indicates a need for further teaching? a) "This medication is taken by mouth." b) "This should be given with food to avoid upsetting his stomach." c) "I should monitor for signs of easy bruising or bleeding gums." d) "This can be taken with other medications we have at home that didn't require a prescription."

"This can be taken with other medications we have at home that didn't require a prescription." Correct Explanation: The nurse must emphasize that the parents should read closely labels of over-the-counter medications they already have or purchase. Some may contain ibuprofen or other nonsteroidal anti-inflammatory drugs, and if given in conjunction with ibuprofen may lead to overdose. The other statements are correct.

D. provide anesthesia to the wound. The combination of lidocaine, adrenaline, and tetracaine provides anesthesia within 10 to 15 minutes of application. LAT does not have a cleansing effect. LAT has no effect on scab formation. LAT has no antibacterial effect.

A child is being seen in the emergency department with multiple facial abrasions and lacerations. The combination agent lidocaine, adrenaline, and tetracaine (LAT) is applied topically to the wounds. The purpose of this combination therapy is to: A. cleanse the wound. B. promote scab formation. C. prevent infection of the wound. D. provide anesthesia to the wound.

The nurse administered morphine intravenously to a four-year-old postoperative patient. Thirty minutes later, the nurse assesses the child. Which assessment finding requires further evaluation? A. Pulse decreased from 136 to 104 B. Blood pressure dropped from 110/72 to 90/55 C. Respiratory rate went from 42 to 16 D. Child pulls away from nurse who wants to assess surgical site

C. Respiratory rate went from 42 to 16 This respiratory rate is on the low side for the age group and requires further evaluation to determine if the child's respirations are being depressed. The normal pulse rate for children two to five years of age is between 70 and 120. 90/55 is a normal finding for this age group. The child pulling away from the nurse is normal behavior for a four-year-old child.

Which of the following medication types lack a ceiling effect, and therefore are prescribed in initial doses that must be titrated to achieve pain relief while managing side effects? a) Ibuprofen b) Morphine c) Tylenol d) Aspirin

Morphine Correct Explanation: Recommended doses of analgesics and co-analgesics that have a ceiling of effect generally reflect the dose required to achieve therapeutic analgesic blood levels; whereas recommended doses of opioids and co-analgesics that lack a ceiling of effect are considered initial dosages. These recommended doses are based on empiric evidence and clinical experience and must be titrated to achieve optimal analgesia while minimizing analgesic-related side effects. Ibuprofen, Tylenol, and aspirin are not opioids and therefore do not have a ceiling.

The emergency department nurse is reviewing the medical record of a recently admitted infant and notes that the child's pain score is 14. The nurse realizes that the infant was assessed using which of the following? a) Riley Infant Pain Scale b) The FLACC behavioral scale c) The Pain Observation Scale for Young Children d) The Neonatal Infant Pain Scale

Riley Infant Pain Scale Explanation: Used was the Riley Infant Pain Scale because the maximum score that can be obtained is 18. The maximum score for the FLACC behavioral scale is 10. The maximum score for the Neonatal Infant Pain Scale and the Pain Observation Scale for Young Children is 7. However, the pain scale used should have been identified in the child's medical record.

Which of the following includes negative effects that chronic pain can have on the pediatric population? a) Weight loss, increased blood pressure, and increased heart rate b) Increased blood pressure, increased heart rate, and sleep disturbances c) Increased appetite, sleep disturbances, and irritability d) Sleep disturbances, exhaustion, irritability, mood disturbances, and depression

Sleep disturbances, exhaustion, irritability, mood disturbances, and depression Correct Explanation: The effects of chronic pain on the child may include sleep disturbances, exhaustion, irritability, mood disturbances, and depression. Heart rate and blood pressure typically return to normal values with chronic pain.

The nurse is working with a 5-year-old boy who must receive repeated intravenous injections as part of his treatment. He hates the injections, however, and is frightened whenever he sees the syringe and needle. In an attempt to overcome this fear, the nurse holds the syringe up for him to see and tells him, "This looks kind of like a space rocket, don't you think? Here comes the space rocket—it needs to refuel." Which pain management technique is the nurse using here? a) Thought stopping b) Hypnosis c) Biofeedback d) Substitution of meaning

Substitution of meaning Correct Explanation: Substitution of meaning or guided imagery is a distraction technique to help a child place another meaning (a nonpainful one) on a painful procedure. As an example, a child could imagine a venipuncture needle as a silver rocket ship probing the moon or a submarine diving under the water to escape a torpedo just in time. Thought stopping is a technique in which children learn to stop anxious thoughts by substituting a positive or relaxing thought in its place. Hypnosis involves the child entering a trance-like state to effectively avoid sensing pain. Biofeedback is based on the theory people can regulate internal events such as heart rate and pain response in response to a stimulus. A biofeedback apparatus is used to measure muscle tone or the child's ability to relax.

The nurse working in pediatrics is aware of the special needs of children related to pain assessment. Which of the following is the highest priority for the nurse to consider when completing a pain assessment? a) Developmental age of child b) Chronological age of child c) Reason for the pain d) Pain medication used and last dose administered

Developmental age of child Correct Explanation: While all of the options are important for assessing pain in children, the highest priority to provide an appropriate pain assessment is knowing the developmental age of the child. The chronological and developmental ages may differ and care needs to be based on both, but the type of pain assessment tool used will be based on the developmental age.

The nurse is caring for a 12-year-old cerebral palsy patient who is unable to communicate verbally. Which of the following pain assessment tools is the most appropriate for the nurse to use when assessing pain in this patient? a) Adolescent pediatric pain tool (APPT) b) Numeric rating scale c) Pain diary d) Face, leg, activity, cry, and consolability (FLACC) descriptors

Face, leg, activity, cry, and consolability (FLACC) descriptors Correct Explanation: If the child has cognitive deficits, use tools for pain assessment that are appropriate for developmental level. Do not use chronologic age as a basis for pain assessment. When the child is unable cognitively to communicate pain, assessment methods used for infants may be employed, such as a FLACC scale. Use of a pain diary requires handwriting skills and use of a numeric rating scale or APPT scale requires verbal and/or cognitive skills.

A nurse is caring for a boy preparing to undergo a dressing change. Which of the following statements by the father lets the nurse know that the child's pain experience is at risk of being intensified? a) "Let's think about something you really like." b) "I will be here for you the whole time." c) "You can hold my hand if you want to." d) "I hope that you will be a brave boy and not cry."

"I hope that you will be a brave boy and not cry." Correct Explanation: Parents can increase or decrease the child's ability to handle a situation. Showing disapproval about crying and expecting the boy to be brave may intensify the pain experience and be beyond the child's coping capabilities. Reacting to the child's pain in an accepting manner and offering comfort measures helps the child cope

2. The nurse is caring for a 6-year-old girl who had surgery 12 hours ago. The child tells the nurse that she does not have pain, but a few minutes later tells her parent that she does. Which of the following should the nurse consider when interpreting this? a. Truthful reporting of pain should occur by this age. b. Inconsistency in pain reporting suggests that pain is not present. c. Children use pain experiences to manipulate their parents. d. Children may be experiencing pain even though they deny it to the nurse.

ANS: D d. Children may deny pain to the nurse because they fear receiving an injectable analgesic or because they believe they deserve to suffer as a punishment for a misdeed. They may refuse to admit pain to a stranger, but readily tell a parent. a and b. These are common fallacies about children and pain. c. Pain is whatever the experiencing person says it is, whenever the person says it exists. Pain would be not questioned in an adult 12 hours after surgery.

4. Physiologic measurements in children's pain assessment are: a. the best indicator of pain in children of all ages. b. essential to determine whether a child is telling the truth about pain. c. of most value when children also report having pain. d. of limited value as sole indicator of pain.

ANS: D d. Physiologic manifestations of pain may vary considerably, not providing a consistent measure of pain. Heart rate may increase or decrease. The same signs that may suggest fear, anxiety, or anger also indicate pain. In chronic pain, the body adapts, and these signs decrease or stabilize. a and c. These are of limited value and must be viewed in the context of a pain-rating scale, behavioral assessment, and parental report. b. When the child states that pain exists, it does. That is the truth.

Which of the following scenarios demonstrates the nurse's knowledge when using guided imagery to relieve pain in pediatric patients? a) Leading a 4-year-old male patient to a guided imagery of him being an airplane pilot and flying across the sky b) Leading a 6-year-old female patient in a fairy princess setting where she is the princess and the nurse is the queen c) After achieving a relaxed state, beginning guided imagery of a fun birthday party for a 3-year-old female patient d) After achieving a relaxed state, beginning a guided imagery of walking down a sandy beach and collecting seashells, a favorite activity of the 13-year-old female patient

After achieving a relaxed state, beginning a guided imagery of walking down a sandy beach and collecting seashells, a favorite activity of the 13-year-old female patient Correct Explanation: Imagery begins with achieving a relaxed state. Guide the child to choose a favorite place. When using guided imagery, do not lead the child; let the child become immersed in their personal image and take command of the experience. Guided imagery is not appropriate for preschoolers and toddlers

When preparing to perform an IV insertion, which of the following is the most important nursing intervention? a) Explain the procedure to the child before beginning. b) Apply EMLA cream to two possible insertion sites and wait at least 10 minutes before attempting. c) Obtain an informed consent. d) Explain the need for the IV to the child before beginning.

Apply EMLA cream to two possible insertion sites and wait at least 10 minutes before attempting. Explanation: EMLA cream reduces the pain of IV catheter insertion and reduces the child's distress. Cover two potential sites, in case the first stick is unsuccessful. An explanation of the IV insertion will vary based on the developmental level of the child. It is more important to prevent pain. Informed consent is not necessary for IV insertion.

During the nurse's initial assessment of a school-age child, the child reports a pain level of 6 out of 10. The child is lying quietly in bed watching television. The nurse should: A. Reassess the child in 15 minutes to see if the pain rating has changed B. Administer the prescribed analgesic C. Do nothing, since the child appears to be resting D. Ask the child's parents if they think the child is hurting

B. Administer the prescribed analgesic School-age children are old enough to report their pain level accurately. A pain score of six is an indication for prompt administration of pain medication. The child might be trying to be brave or might be lying still because movement is painful. There is no need to reassess, as the child needs pain medication now. The child might be lying quietly because movement increases the pain. The school-age child can answer for herself and does not need the parents to answer for her.

The nurse is caring for a 7-year-old post-op child who is reporting an 8 out of 10 on a pain intensity scale. The child's mother is requesting pain medication. The child received ibuprofen three hours ago. Which of the following is the correct nursing action? a) Contact the physician and request an opioid pain medication. b) Explain to the mother the child cannot receive another dose of ibuprofen for three hours. c) Apologize to the mother and tell her there is nothing you can do at the moment. d) Turn on the television in hopes to distract the child.

Contact the physician and request an opioid pain medication. Correct Explanation: The nurse must advocate for the child. Advocacy may involve convincing a parent that opioids are appropriate for the situation or consulting with the prescriber regarding an ineffective medication regimen. Explaining to the mother that the child cannot receive any more pain medication is ineffective and does not advocate for the child in pain. Turning on the television is not a bad idea; however, it is not the priority. It is not appropriate to apologize. The nurse can do something. Contacting the physician to request more medication is in the nurse's power.

When educating a parent how to support their child while experiencing a painful procedure, which of the following is the best information for the nurse to convey to the parents? a) Encourage the parents to focus on the time remaining during the procedure. b) Encourage the parents to praise the child at each step of the painful procedure. c) Have the parents apologize to the child. d) Explain in detail the role of the parent as a coach and emphasize the coping plan.

Explain in detail the role of the parent as a coach and emphasize the coping plan. Correct Explanation: Break down complex procedures into specific steps and reinforce coping strategies for each distinct task. Model, or demonstrate, coping behaviors. Detail parents' coaching role and reinforce the need to emphasize the coping plan, rather than praising the child or apologizing for the pain. The parents should be focused on the coping plan, not on the time remaining

Which of the following nursing interventions demonstrates proper use of cutaneous stimulation to relieve pain with pediatric patients? a) Use of a heat pack after abdominal surgery for a 2-week-old infant with necrotizing enterocolitis b) Use of a cold pack for the treatment of cellulitis on an extremity c) Gently massaging a pre-term infant's leg for two minutes prior to obtaining a blood sample from a heel stick d) Use of a cold pack for 20 minutes to achieve a muscle temperature of 40 °C

Gently massaging a pre-term infant's leg for two minutes prior to obtaining a blood sample from a heel stick Correct Explanation: Gentle massage of the leg for two minutes prior to heel stick may decrease pain response in pre-term infants. Use of heat or cold therapy is contraindicated in infants, who are more prone to thermal injuries. Ice packs should not be used for longer than 15 minutes at a time. Heat is most effective in relieving pain from inflammation and spasm.

The nurse is caring for a 6-year-old child with burns on both hands. Which of the following pain assessment techniques is the most accurate for this patient? a) Assessment of the burns b) Behavior c) Obtain a self-report d) Physiological indicators

Obtain a self-report Correct Explanation: A proposed hierarchy of assessment techniques can guide nurses in determining the presence and intensity of patients' pain (Manworren, Paulos, and Pop, 2004). Ranked in order by their importance and reliability for assessing pain are self-report, presence of pathology or a condition associated with pain, behavior, proxy ratings, and finally, physiological indicators of pain.

Adam is a 14-year-old post-op scoliosis rod surgery patient. As the nurse enters the room, she observes the 3-year-old sibling with the PCA button in his hand, pressing the button multiple times. The patient and his mother are both asleep. Which of the following is the correct term for this action? a) PCA by toddler b) PCA by proxy c) Incident report d) PCA by accident

PCA by proxy Correct Explanation: PCA by proxy is the correct term used when an unauthorized person activates the dosing mechanism.

The nurse is caring for a 6-year-old patient with chronic pain. Which of the following is the best choice of pain management for this patient? a) Rectal morphine PRN b) PO morphine PRN c) IM morphine PRN d) IV morphine PRN

PO morphine PRN Correct Explanation: The preferred route of medication administration for children is the oral route. The oral is convenient, and relatively steady blood levels of drug can be achieved to ensure steady and consistent pain control, with few peaks and valleys of pain and side effects

A nurse is attempting to assess the extent of an injury a 5-year-old boy sustained when he fell down a flight of stairs. The child is visibly upset but capable of communicating. Which of the following pain scales would be most appropriate to use in this situation? a) FLACC pain assessment tool b) Poker chip tool c) Pain experience inventory d) CRIES inventory

Poker chip tool Explanation: The Poker Chip Tool uses four red poker chips placed in a horizontal line in front of the child. Each chip represents a different level of pain. The technique can be used with children as young as 4 years of age, provided the child has some concept of "more or less." The Pain Experience Inventory is designed to elicit the terms a child uses to denote pain and what actions a child thinks will best alleviate pain. If possible, it should be used before the child has pain. The CRIES inventory is a 10-point scale named for five physiological and behavioral variables commonly associated with neonatal pain, and thus is not age-appropriate in this case. The FLACC Pain Assessment Tool is a scale by which health care providers can rate a young child's pain when a child cannot give input, such as during circumcision.

The nurse is caring for several infants who require heel sticks to obtain a blood sample. Which infant is likely to experience the greatest intensity of pain from this procedure? a) 10-month-old b) 2-month-old c) Neonate d) Preterm infant

Preterm infant Explanation: Research suggests that preterm infants experience pain at a greater intensity than older children or even adults. The reason for this may be that the inhibitory mechanisms higher in the central nervous system have not had time to develop.

What behavioral responses to pain would a nurse observe from an infant younger than age 1? a) Passive resistance, clenching fists, and holding body rigid b) Reflex withdrawal to stimulus and facial grimacing c) Localized withdrawal and resistance of the entire body d) Low frustration level and striking out physically

Reflex withdrawal to stimulus and facial grimacing Explanation: Infants younger than age 1 become irritable and exhibit reflex withdrawal to the painful stimulus. Facial grimacing also occurs. Localized withdrawal is experienced by toddlers ages 1 to 3 in response to pain. The nurse would observe passive resistance in school-age children. Preschoolers show a low frustration level and strike out physically

The nurse is working with the mother of a 6-year-old girl to think of an effective means of distracting the girl from a painful procedure that she will shortly undergo. To be effective, the distraction technique must have which of the following characteristics? a) Requires interaction with another person b) Makes the child laugh c) Refers to some past positive experience of the child d) Requires concentration of the child

Requires concentration of the child Correct Explanation: When helping parents choose a distraction technique such as blowing soap bubbles with their child, be certain they do not interpret "distraction" as just talking to the child or suggesting a video game to divert attention. Although these are distractions, a distraction activity must require concentration; simple distractions can allow pain to break through. The other answers listed are not necessary of distraction techniques.

The nurse is caring for a patient receiving opioid medication for the treatment of postoperative pain. Which of the following are common side effects that the nurse should observe the patient for? a) Respiratory depression, diarrhea, and hypotension b) Hypotension, nausea and vomiting, and diarrhea c) Respiratory depression, constipation, and pruritis d) Constipation, hypertension, and disorientation

Respiratory depression, constipation, and pruritis Correct Explanation: Nausea and vomiting, pruritis, sedation, respiratory sedation, constipation, and urinary retention are common side effects of opioid medications, so answer B is correct. Hypotension, hypertension, diarrhea, and disorientation are not common side effects of opioid medication.

D. facial expression of discomfort. A. Respiratory pattern may be markedly variable in an infant in pain and thus is not a consistent indicator of pain. B. Heart rate may initially decrease in some infants with pain and then increase; thus it is not a consistent indicator of pain. C. Clenching the teeth and lips are signs of pain often assessed in the toddler, not the infant. D. Facial expression of discomfort is the most consistent behavioral manifestation of pain in infants.

The most consistent indicator of pain in infants is: A. increased respirations. B. increased heart rate. C. clenching the teeth and lips. D. facial expression of discomfort.

A. Request a psychological consultation. A psychological consultation will assist the child in verbalizing fears. This age-group is very concerned with physical appearance. The psychologist can help integrate the issues the child is facing. It is likely that the child is having pain but not acknowledging the pain. Speaking with a psychologist might assist the child in relaying fear and pain. If the child is feeling pain, the nurse should not offer praise for hiding the pain. The nurse should encourage the child to speak up during painful episodes so that the pain can be managed appropriately. Bravery may not be an effective coping strategy if the child is in severe pain.

The nurse is caring for a 12-year-old child who sustained major burns when putting charcoal lighter on a campfire. The nurse observes that the child is "very brave" and appears to accept pain with little or no response. What is the most appropriate nursing action? A. Request a psychological consultation. B. Ask why the child does not have pain. C. Praise the child for the ability to withstand pain. D. Encourage continued bravery as a coping strategy.

C. requires astute nursing assessment and management. Because the child cannot communicate pain through one of the standard pain rating scales, the nurse must focus on physiologic and behavioral manifestations to accurately assess pain. Pain can occur in the comatose child. The child can be in pain while comatose. The family can provide insight into the child's different responses, but the nurse should be monitoring physiologic and behavioral manifestations.

The nurse is caring for a comatose child with multiple injuries. The nurse should recognize that pain: A. cannot occur if a child is comatose. B. may occur if a child regains consciousness. C. requires astute nursing assessment and management. D. is best assessed by family members who are familiar with the child.

A. this practice is unjustified and unethical. A. Placebos should never be given by any route in the assessment or management of pain. B. Placebos should never be given as a means to determine whether pain is real. Individuals respond differently to placebos; thus the patient's response may not be an accurate measure of pain. C. Response to a placebo is not a measure of the origin of pain and should never be used as a means of assessing pain. D. Response to a placebo is not a measure of the origin of pain and should never be used as a means of assessing pain.

The nurses caring for a child are concerned about the child's frequent requests for pain medication. During a team conference, a nurse suggests that they consider administering a placebo instead of the usual pain medication. This decision should be based on knowledge that: A. this practice is unjustified and unethical. B. this practice is effective in determining whether a child's pain is real. C. the absence of a response to a placebo means the child's pain has an organic basis. D. a positive response to a placebo will not occur if the child's pain has an organic basis.

The nurse is caring for a child who has received post-operative epidural analgesia. What is the priority nursing assessment? a. urinary retention b. pruritus c. Nausea and vomiting d. Respiratory depression

d. respiratory depression

C. Repeatedly stating, "You're hurting me." D. Clinching fists and tensing arms in anticipation. Developmental characteristics of the adolescent's response to pain include: less vocal protest; less motor activity; more verbal expressions, such as "It hurts" or "You're hurting me"; and increased muscle tension and body control. Stating "You're hurting me" and muscle tension are expected responses to pain for the adolescent.

When changing a dressing on the leg of a 16-year-old patient who suffered second degree burn injuries, the nurse expects to observe which characteristics of pain expression? (Select all that apply.) A. Stomping feet on the ground and screaming, "No" B. Attempting to move leg out of reach of the nurse. C. Repeatedly stating, "You're hurting me." D. Clinching fists and tensing arms in anticipation. E. Scooting away and asking parents to stop the nurse.

The nurse is preparing to assess the pain of a developmentally and cognitively delayed 8 year old. Which of the pain rating scales should the nurse choose? a. FACES pain rating scale b. Word Graphic Rating Scale c. Adolescent Pediatric Pain tool d. Visual analog and Numerical scales

a. FACES

The nurse is providing post-surgical care for a 5 year old. The nurse knows to avoid which of the following questions when assessing the child's pain level? a. would you say that the pain you are feeling is sharp or dull? b. Would you point to the cartoon face that best describes your pain? c. Would you point to the spot where your pain is? d. Would you please show me with photograph and number best describes your hurt?

a. Would you say that the pain you are feeling is sharp or dull?

A newborn who is suspected of having leukemia is being prepared for bone marrow aspiration. The newborn's mother asks whether any type of sedation or anesthesia will be used. Which of the following should the nurse say in response? a) "Because this is a painful procedure, your child will receive conscious sedation to alleviate pain." b) "Because newborns lack memory, no sedation is needed." c) "Because myelination of the peripheral nerves is incomplete at this age, the newborn cannot experience pain. Thus, no sedation is needed." d) "Because of the risks involved, your child will not be receiving any sedation."

"Because this is a painful procedure, your child will receive conscious sedation to alleviate pain." Correct Explanation: In the past, it was believed infants do not feel pain because of incomplete myelination of peripheral nerves. Evidence-based practice has shown this not to be true as myelination is not necessary for pain perception. A second argument in the past against needing to provide pain relief for infants was that they have no memory. It can be shown, however, physiologic changes occur with pain even in preterm infants, so even with a lack of memory, it is clear pain is experienced. Sedation does not typically involve risk high enough to forgo it before a painful procedure.

When providing education to a mother regarding pain management for a toddler with otitis media, which statement by the mother indicates further teaching needs for this parent? a) "I should give my toddler one baby aspirin." b) "Use of infant Tylenol is good for my toddler." c) "I don't have to give my child pain medication unless he needs it." d) "Ibuprofen can be purchased over the counter to use if my toddler needs it."

"I should give my toddler one baby aspirin." Correct Explanation: Children should not use acetylsalicylic acid for routine pain management because of the increased risk of Reye syndrome. The use of the other options is appropriate and all indicate an understanding of pain management for this child.

The nurse is preparing a 6-year-old for a venipuncture. The boy appears anxious and is crying. How can the nurse foster feelings of control to help minimize his anxiety about the procedure? a) "Mrs. Jones, why don't you have him sit on your lap?" b) "See how fast you can make this pinwheel whirl." c) "What questions do you have about what I am doing?" d) "Pick your favorite Band-Aid and show me which arm to use."

"Pick your favorite Band-Aid and show me which arm to use." Correct Explanation: Allowing the child options related to the style of the Band-Aid and the extremity to use gives the child some control over the happenings. Offering a pinwheel is a distraction technique. Encouraging the parent to hold the child during the procedure promotes feelings of security. Encouraging the child or parents to ask questions facilitates communication.

The mother with a child following abdominal surgery holds his hand and smoothes his hair. When the nurse appears to administer a scheduled analgesic, the mother says she believes the child has been in pain the last hour or more. The nurse's best response is: a) "He looks comfortable to me and was sleeping each time I checked. The medication given earlier seems adequate." b) "This is the medication he gets every 4 to 6 hours. It will control his pain for that period of time." c) "We will check every 4 hours to see if he needs the pain medicine." d) "His vital signs are stable, telling me he must be comfortable." e) "Please tell me and all of the nurses when you believe he is in pain."

"Please tell me and all of the nurses when you believe he is in pain." Correct Explanation: Having the mother share her assessments is very helpful. She knows the child the best. The statements about the medication and checking every 4 hours may be true but do not acknowledge the mother and the importance of her input. Looking comfortable (stillness) and even sleeping can be a coping strategy used by the child. Stable vital signs can be misleading. (Continually elevated signs can be interpreted as stable.) Many events can raise vital signs, including anxiety or happy excitement. Physiologic signs should be interpreted with care and combined with behavioral signs of pain.

The father of a periorbital cellulitis patient comes to the nurses' station and asks to speak with the nurse. He demands to have more medication for his child because "Whatever you gave him 20 minutes ago just isn't working!" Which of the following is the best response? a) "You need to be patient and wait for the medicine to work." b) "There is nothing I can do for you right now. The next dose isn't due for another 6 hours." c) "Please return to your room and I will be with you in a moment." d) "The oral pain medication we gave 20 minutes ago typically takes about 45 minutes to take effect and peaks in about 1 to 2 hours. I will be back in 20 minutes to check the pain level again."

"The oral pain medication we gave 20 minutes ago typically takes about 45 minutes to take effect and peaks in about 1 to 2 hours. I will be back in 20 minutes to check the pain level again." Correct Explanation: Oral pain medicine takes about 45 minutes to digest and take effect; then it peaks in 1 to 2 hours. Before dosing again, give the medicine some more time to work and then reassess the pain. Stating there is nothing you can do is unhelpful. Telling a parent to be patient will not diffuse the parent's anger. Putting the parent off will only intensify the situation.

Heather is a 9-year-old one-day post-op appendectomy patient. She is due to receive a dose of IV morphine. Her mother is at the bedside and asks why Heather needs another dose of morphine when she received one just 4 hours ago. Which statement shows the nurse's understanding of pain management? a) "Do you want your daughter to be in pain?" b) "I am just following the doctor's orders." c) "The physician has ordered morphine to be given around the clock in order to keep on top of your daughter's postoperative pain." d) "I am certain your daughter will not become an addict in just one day."

"The physician has ordered morphine to be given around the clock in order to keep on top of your daughter's postoperative pain." Correct Explanation: It is important to provide pain medication around the clock to manage pain more effectively and avoid peaks and cycles of pain. The response "I am just following the doctor's orders" does not answer the question. The mother never expressed concern about her daughter becoming an addict. Asking if the mother wants her daughter to be in pain is judgmental and inappropriate.

The nurse is preparing to use biobehavioral interventions to control pain. Which of the following statements to the patient's mother demonstrates the nurse's knowledge of this approach? a) "How about we read a book now so we won't have to use any pain medications." b) "Why aren't you trying to distract your child by reading a book together?" c) "What is your child's favorite movie? We have many movies here we can play in the room to focus on something other than the pain." d) "Why don't we turn on the television so you won't focus so much on your child's pain?"

"What is your child's favorite movie? We have many movies here we can play in the room to focus on something other than the pain." Correct Explanation: Watching a favorite movie can help distract a child from pain and place focus on the movie. Biobehavioral interventions should be used along with pharmacological interventions, not in place of them. Choice C focuses on the parent, not the patient. Choice D is placing blame and inappropriate

The nurse is caring for a 15-year-old patient involved in a motor vehicle accident (MVA). The patient is receiving patient-controlled analgesia via an epidural for pain in the extremities due to bilateral compound leg fractures. Which of the following statements is correct when teaching this patient how to manage pain in this manner? a) "Press the red button when you experience high levels of pain." b) "You might experience decreased sensation and ability to move your legs while using this route of medication." c) "You won't need to keep the cardiac monitor in place once you have been on this medication for a few hours." d) "Tell your parents they need to press the button for you if you fall asleep."

"You might experience decreased sensation and ability to move your legs while using this route of medication." Correct Explanation: Epidural analgesia can cause decreased sensation and ability to move the lower extremities. PCA medication should only be administered by the patient unless otherwise indicated. A cardiac monitor along with frequent physical assessment is a good way to monitor for respiratory depression. Removing the monitor after a few hours would be contraindicated. The patient should be taught to press the button before severe levels of pain are reached to decrease peaks and valleys in pain control.

The nurse caring for a 28-week pre-term infant prepares to obtain a capillary blood specimen for a routine bilirubin test via heel stick. The parents are at the bedside and ask the nurse if their infant will feel pain when he is stuck. Which of the following is the best response for the nurse to make? a) "Your son will experience pain for a brief moment when his heel is stuck." b) "Your son will not feel any pain since his neurological system is immature." c) "Your son will feel pain, but he will never remember it." d) "You shouldn't be concerned about a small routine procedure like a heel stick."

"Your son will experience pain for a brief moment when his heel is stuck." Correct Explanation: Neuroanatomical and neuroendocrine components of the pain pathway are sufficiently developed in the neonate to allow the transmission and perception of pain. Ample evidence indicates that both term and pre-term neonates have the capacity to experience and remember pain much like older children and adults do. Telling a parent not to be concerned about a routine procedure diminishes their feelings and is not appropriate

B. greater than the IV dose. A. Oral morphine is not as effective at the same dose as IV morphine. B. When the route of morphine administration is changed from IV to PO (by mouth), it is essential that the dosage be increased to achieve an equianalgesic effect. C. The dosage of morphine is increased, not decreased, when the administration route changes from IV to PO. D. The dosage of morphine is increased, not decreased, when the administration route changes from IV to PO.

A child who has been receiving morphine intravenously will now start receiving it orally. The nurse should anticipate that, to achieve equianalgesia (equal analgesic effect), the oral dose will be: A. the same as the intravenous (IV) dose. B. greater than the IV dose. C. one half of the IV dose. D. one fourth of the IV dose.

D. large doses of opioids are justified when there are no other treatment options. A. Continuing studies report that children are consistently undermedicated for pain. B. The dosage of opioids is titrated to relieve pain, not cause death. C. Addiction refers to a psychologic dependence on the narcotic medication, which does not occur in terminal care. D. Large doses of opioids may be needed because the child has become physiologically tolerant to the drug, requiring higher doses to achieve the same degree of pain control. Pain is considered the fifth vital sign, and management of pain is critical to treatment of a child with bone cancer.

A child who is terminally ill with bone cancer is in severe pain. Nursing interventions should be based on knowledge that: A. children tend to be overmedicated for pain. B. giving large doses of opioids causes euthanasia. C. narcotic addiction is common in terminally ill children. D. large doses of opioids are justified when there are no other treatment options.

Which of the following statements is the goal of distraction techniques used to control pain? a) The goal of distraction interventions is to include the child in purposeful behaviors in order to have the child feel more in control of the situation. b) The goal of distraction interventions is to reduce parental anxiety by entertaining the child. c) The goal of distraction interventions is to relieve pain completely through the use of specific techniques to divert the child's attention away from the pain, thereby eliminating the need for pain medication. d) A goal of distraction interventions is to divert the child's attention away from the pain through controlled, purposeful behaviors.

A goal of distraction interventions is to divert the child's attention away from the pain through controlled, purposeful behaviors. Correct Explanation: The goal of distraction interventions is to divert the child's attention away from the pain through controlled, purposeful behaviors. Distraction interventions are used in conjunction with pain medications to reduce pain. Giving the patient choices—not using distraction techniques—enables the patient to have a greater sense of control. The goal of distraction interventions is not parent focused and the purpose is entertainment for the child.

The clinical nurse specialist is concerned about children's reactions to painful invasive procedures such as intravenous starts. The nurse has decided to use distraction as a means to comfort the school-age child. Depending on the age of the school-age child, which technique might the nurse use to distract the child? (Select all that apply.) A. Blowing bubbles B. Music therapy C. Guided imagery D. Hypnosis E. Sucrose solution

A. Blowing bubbles B. Music therapy C. Guided imagery Blowing bubbles or popping bubbles can be a distraction for a young school-age child. Listening to music or singing can be used as distraction for this age group. Guided imagery is a means of encouraging relaxation and mental images to manage pain. Under hypnosis, the child is an altered state of awareness; this is not a form of distraction. Sucrose solution is used for infants up to 12 months of age. It has not been found to be effective in the school-age child.

A nurse is taking care of a patient in the ICU who has been on opioids for an extended period of time. The nurse understands that the child has to slowly wean from the medication over a period of time. While weaning, the nurse will observe the child for symptoms of too rapid withdrawal, including: A. Hyperactive deep tendon reflexes, vomiting, and abdominal cramps B. Bradycardia and pallor C. Decreased blood pressure and drowsiness D. Voracious appetite and hypotonicity

A. Hyperactive deep tendon reflexes, vomiting, and abdominal cramps These are symptoms of withdrawal resulting from reducing the dose too quickly. A child who is being withdrawn from opioids too quickly will be tachycardic and have hot flashes and sweating. The child who is being withdrawn from opioids too quickly will be hypertensive and wakeful. Nausea, abdominal pain, diarrhea, and hypertonicity would be symptoms of withdrawal.

The nurse is caring for a four-year-old child who is intellectually disabled and is scheduled for surgery tomorrow. The nurse wants to plan postoperative care and pain relief. The nurse will determine the best pain assessment tool by observing the child's: (Select all that apply.) A. Language skills B. Understanding of the concept of more and less or otherwise has the ability to quantify pain C. Ability to sit for a ten minute evaluation D. Ability to perceive pain E. Ability to understand pain

A. Language skills B. Understanding of the concept of more and less or otherwise has the ability to quantify pain In order to report pain, the child needs adequate verbalization skills to communicate to the nurse. The child who understands more or less can be given a three-option pain scale. The child who cannot understand more or less may need a behavioral pain scale. The assessment does not require the child to sit still. Children perceive pain. The issue is if the nurse can recognize the child's pain. Children do not need to understand pain in order to feel pain.

The nurse is taking care of a seven-year-old child who is postoperative. The child's mother requests that the child not receive narcotics in the postoperative period because she is afraid the child will become addicted. The nurse would explain to the mother that children who do not receive adequate pain control will be at risk for: A. Respiratory complications B. Urinary complications C. Cardiac complications D. Bowel complications

A. Respiratory complications The child with acute postoperative pain takes shallow breaths and suppresses coughing to avoid more pain. These self-protective actions increase the potential for respiratory complications. Uncontrolled pain does not usually lead to urinary complications. Uncontrolled pain does not lead to cardiac complications. Uncontrolled pain does not frequently lead to bowel complications.

The nurse is working in a pediatric surgical unit. The nurse would expect that patient-controlled analgesia would be most appropriate for which patient? A. Twelve-year-old who is postoperative for spinal fusion for scoliosis B. Ten-year-old who has a fractured femur and concussion from a bike accident C. Five-year-old who is postoperative for tonsillectomy D. Developmentally delayed 16-year-old who is postoperative for bone surgery

A. Twelve-year-old who is postoperative for spinal fusion for scoliosis Patient-controlled analgesia is most appropriate in children five years old and older. The child must be able to press the button and understand that she will receive pain medicine by pushing the button. Children who have suffered head trauma would not be candidates for PCA. PCA generally is prescribed for clients who will be hospitalized for at least 48 hours. Children who are developmentally delayed would not be candidates for PCA.

5. Nonpharmacologic strategies for pain management: a. may reduce pain perception. b. make pharmacologic strategies unnecessary. c. usually take too long to implement. d. trick children into believing they do not have pain.

ANS: A a. Nonpharmacologic techniques provide coping strategies that may help reduce pain perception, make the pain more tolerable, decrease anxiety, and enhance the effectiveness of analgesics. b. Nonpharmacologic techniques should be learned before the pain occurs. With severe pain, it is best to use both pharmacologic and nonpharmacologic measures for pain control. c. The nonpharmacologic strategy should be matched with the child's pain severity and taught to the child before the onset of the painful experience. d. Some of the techniques may facilitate the child's experience with mild pain, but the child will still know the discomfort was present.

8. The nurse is caring for a child receiving intravenous (IV) morphine for severe postoperative pain. The nurse observes a slower respiratory rate, and the child cannot be aroused. The most appropriate management of this child is for the nurse to do which of the following? a. Administer naloxone (Narcan). b. Discontinue IV infusion. c. Discontinue morphine until child is fully awake. d. Stimulate child by calling name, shaking gently, and asking to breathe deeply.

ANS: A a. The management of opioid-induced respiratory depression includes lowering the rate of infusion and stimulating the child. If the respiratory rate is depressed and the child cannot be aroused, then IV naloxone should be administered. The child will be in pain because of the reversal of the morphine. b and c. The morphine should be discontinued, but naloxone is indicated if the child is unresponsive. d. The child is unresponsive. Naloxone is indicated.

1. Which of the following factors is an important consideration in understanding the pain experience in children? a. Children cannot tell where they hurt. b. Children may not admit having pain. c. Narcotics are dangerous drugs for children. d. Children's sensitivity to pain is less than that of adults.

ANS: B b. Children may not admit having pain to avoid an injection. With constant pain, children may not realize how much they are hurting and believe that adults will know how they are feeling. a. By age 4 years, children can accurately point to the body area or mark the painful area on a drawing. c. Narcotics are no more dangerous for children than they are for adults. d. Children have the same sensitivity to pain as adults have.

6. Which of the following drugs is usually the best choice for patient-controlled analgesia (PCA) for a child in the immediate postoperative period? a. Codeine b. Morphine c. Methadone d. Meperidine

ANS: B b. The most commonly prescribed medications for PCA are morphine, hydromorphone, and fentanyl. a. Parenteral use of codeine is not recommended. c. This is not available in parenteral form in the United States. d. Meperidine is not used for continuous and extended pain relief.

7. A lumbar puncture is needed on a school-age child. The most appropriate action to provide analgesia during this procedure is to apply: a. TAC (tetracaine-adrenaline-cocaine) 15 minutes before procedure. b. transdermal fentanyl (Duragesic) patch immediately before procedure. c. EMLA (eutectic mixture of local anesthetics) 1 hour before procedure. d. EMLA (eutectic mixture of local anesthetics) 30 minutes before procedure.

ANS: C c. EMLA is an effective analgesic agent when applied to the skin 60 minutes before a procedure. It eliminates or reduces the pain from most procedures involving skin puncture. a. TAC provides skin anesthesia about 15 minutes after application to nonintact skin. The gel can be placed on wound for suturing. b. Transdermal fentanyl patches are useful for continuous pain control, not rapid pain control. d. For maximal effectiveness, EMLA must be applied approximately 60 minutes in advance.

3. Kyle, age 6 months, is brought to the clinic. His parent says, "I think he hurts. He cries and rolls his head from side to side a lot." This most likely suggests which of the following features of pain? a. Type b. Severity c. Duration d. Location

ANS: D d. The child is displaying a local sign of pain. Rolling the head from side to side and pulling at ears are indicative of pain in the ear. a, b, and c. The child's behavior indicates the location of the pain. The behavior does not provide information about the type, severity, or duration.

A 7-year-old boy tells the nurse that his head sometimes hurts after he eats ice cream. The nurse recognizes that this type of pain is which of the following? a) Acute visceral pain b) Chronic somatic pain c) Chronic cutaneous pain d) Acute referred pain

Acute referred pain Correct Explanation: Acute pain means sharp pain, as is the case in this scenario. It generally occurs abruptly after an injury. The pain of a pin prink is an example. Chronic pain is pain that lasts for a prolonged period or beyond the time span anticipated for healing. Referred pain is pain that is perceived at a site distant from its point of origin. In this case, the typical ice cream "brain freeze" is a headache that results from the contact of the cold ice cream with the digestive tract. Cutaneous pain is pain that arises from superficial structures such as the skin and mucous membrane. A paper cut is an example. Somatic pain is pain that originates from deep body structures such as muscles or bones. The pain of a sprained ankle is somatic pain. Visceral pain involves sensations that arise from internal organs such as the intestines. The pain of appendicitis is visceral pain.

The nurse is caring for a burn patient with orders for oral ibuprofen and morphine PRN to control pain. Which of the following nursing interactions is the most beneficial for the nurse to implement fpr pain management? a) Give morphine as little as possible to prevent unwanted side effects. b) Give medication when the patient asks for it. c) Give only the ibuprofen until the patient complains of higher pain levels. d) Alternate these medications around the clock to diminish peaks and valleys in pain control.

Alternate these medications around the clock to diminish peaks and valleys in pain control. Correct Explanation: Pain is best managed by a proactive, preemptive approach. Anticipating and treating pain is much more effective and humane than trying to manage pain once it is present. PRN administration of pain medication tends to propagate a pain cycle with peaks (side effects like sedation) and troughs (pain) of drug action. If pain is present or anticipated for most of the day, medications must be scheduled and administered around the clock (ATC), with additional doses of analgesics available for prompt relief of breakthrough pain.

B. the scale can be used with most children, including those as young as 3 years old. A. The child points at the face that best describes the pain being experienced. B. The FACES scale has been validated for children as young as 3 years old to rate pain. C. The scale is useful for all ages above 3 years, including adults. D. The scale does not have a means of assessing physiologic data.

An important consideration when using the FACES pain rating scale with children is: A. that children color the face with the color they choose to best describe their pain. B. the scale can be used with most children, including those as young as 3 years old. C. the scale is not appropriate for use with adolescents. D. the scale is useful in pain assessment but is not as accurate when assessing physiologic responses.

You plan to apply EMLA cream to decrease the pain of an injection. Which of the following would be the best technique? a) Wipe it off at least 15 minutes before the procedure. b) Apply it at least 1 hour before the procedure. c) Apply it immediately prior to the painful procedure. d) Do not cover it after application to prevent it from discoloring.

Apply it at least 1 hour before the procedure. Correct Explanation: EMLA, a topical anesthetic cream, must be applied at least 1 hour prior to a procedure to be effective. It should be covered after application to prevent the child from tasting it (which could anesthetize the gag reflex) and for maximum absorption.

Conscious sedation is a pain-management technique that is used with children. During conscious sedation for a preschooler, which of the following actions would be most important? a) Assessing vital signs frequently, because they can become depressed b) Keeping the child's head in a dependent position c) Asking the child to periodically count from 1 to 10 d) Keeping the room absolutely quiet so the child can sleep

Assessing vital signs frequently, because they can become depressed Correct Explanation: Conscious sedation is the use of a drug such as pentobarbital sodium to induce a conscious but sleepy state. Vital signs must be monitored closely to be certain the child's vital centers do not become depressed.

In responding to the needs of pediatric patients in pain, the nurse has numerous nonpharmacologic interventions available. These interventions include: (Select all that apply.) A. Regional nerve block B. Cutaneous stimulation C. Application of heat D. Electroanalgesia E. Use of EMLA cream

B. Cutaneous stimulation C. Application of heat D. Electroanalgesia Massage and rubbing of the skin as well as swaddling and kangaroo care are nonpharmacologic means of relieving pain. The use of heat (and cold) may help reduce pain sensations and utilizes no pharmacologic agents. Electrical stimulation to the skin uses the gate control theory to relieve pain. A regional nerve block involves injecting medications in an area that controls pain for a region of the body. It does not provide nonpharmacologic relief. EMLA cream is a mixture of lidocaine and prilocaine that is applied to the intact skin. It is a pharmacologic pain relief method.

The nurse is preparing to assess the post-surgical pain level of a 6 year old. The child has appeared unwilling or unable to accurately report his pain level. Which of the following assessment tools would be most appropriate for this child? a. FACES pain rating scale b. FLACC behavioral scale c. Oucher pain rating d. visual analog and numerical scale

B. FLACC behavioral scale

A nurse is instructing the parents of a toddler on the use of an anesthetic cream in advance of an upcoming procedure the child will have. Which of the following should the nurse mention to them? a) Do not apply the cream until you are at the hospital, a few minutes before the procedure is to begin b) Do not cover the site of the cream application with any kind of dressing c) Administer the cream about 4 hours before the procedure d) Be careful not to let your son remove the dressing, as the cream can cause damage to his eyes if he rubs them

Be careful not to let your son remove the dressing, as the cream can cause damage to his eyes if he rubs them Correct Explanation: To reduce the pain of procedures such as venipuncture, lumbar puncture, and bone marrow aspiration, a local anesthetic cream that contains 4% lidocaine can be used. The cream is applied to the skin, and the site is then covered with an occlusive dressing or plastic wrap to keep young children from wiping away or tasting the cream. The time needed for effect between different brands varies from 30 minutes to 1 hour so must be applied within that time frame before an expected procedure. Caution them not to allow their child to remove the dressing as the cream could anesthetize the gag reflex if eaten or cause eye damage if rubbed into the eyes.

Parents are to bring their kindergarten child to the outpatient department for a venous blood sample. They have EMLA cream to apply at home prior to the procedure and have been shown two areas on the child's arms where they should place the cream. Transportation time is 15 minutes. Their appointment is for 2:45 p.m. At what time should the parents apply the cream and occlusive dressings to both arms? a) Right before leaving home for the clinic b) At 11:45 a.m. c) Between 1:15 p.m. and 1:45 p.m. d) The parents should not apply EMLA cream due to their child's age.

Between 1:15 p.m. and 1:45 p.m. Correct Explanation: The EMLA cream needs to be in place 60 to 90 minutes prior to the procedure in order to attain local analgesia. Applying it at 11:45 a.m. (3 hours prior to the procedure) is necessary for deeper analgesia needed for such things as a lumbar puncture. Right before leaving their home (approximately 20 minutes before the procedure) is effective when Synera is used since it needs to be in place 20 to 30 minutes. EMLA cream is approved for use in children ages 37 weeks and older.

A five-year-old child is being discharged from the outpatient surgical center. Which statement by the parent would indicate the need for further teaching? A. "I will call the office tomorrow if the pain medicine is not relieving the pain." B. "I can expect my child to have some pain for the next few days." C. "Because my child just had surgery today, I can expect the pain level to be higher tomorrow." D. "I will plan to give my child pain medicine around the clock for the next day or so."

C. "Because my child just had surgery today, I can expect the pain level to be higher tomorrow." Increasing pain can be a sign of complication and should be reported to the physician; therefore, the nurse should clarify expectations for pain control. If prescribed medication is not relieving the pain to a satisfactory level, the physician should be notified. This statement indicates the parent understands and does not need additional teaching. The child is expected to have some pain for a few days after surgery. This statement indicates the parent understands the teaching. The child should receive pain medication on a scheduled basis. This statement indicates the parent understands the teaching.

A hospitalized three-year-old needs to have an IV restarted. The child begins to cry when carried into the treatment room by the mother. Which is the most appropriate nursing diagnosis? A. Knowledge deficit of the procedure B. Fear related to the unfamiliar environment C. Anxiety related to anticipated painful procedure D. Ineffective individual coping related to an invasive procedure

C. Anxiety related to anticipated painful procedure This child is not old enough to understand the need for an IV infusion. The question stem indicates that the child has been through this painful procedure before, and his reaction to entering the treatment room is based on anticipation of repeat discomfort. The question stem indicates that the child has been through this before, so Knowledge deficit is not the most appropriate diagnosis. The child's fear is related not to the unfamiliar environment but to the anticipated pain of the IV stick. The child's behavior is appropriate for coping in this age child.

An analgesic is ordered for a post-surgical patient to be given every three to four hours. The nurse knows that a delay in giving the medication will cause a(n): A. Decrease in the chance of withdrawal symptoms B. Decrease in the chance of addiction C. Increase in the chance of breakthrough pain D. Increase in the child's pain tolerance

C. Increase in the chance of breakthrough pain Analgesics may be given on a scheduled basis. Delays in giving analgesics increase the chance of breakthrough pain and the subsequent anticipation of pain. A delay in giving pain medication will not decrease the chance of withdrawal symptoms if the medication is stopped without weaning. Delaying the pain medication will not decrease the chance of addiction or increase the child's pain tolerance.

The nurse is caring for a 12-year-old patient in sickle-cell crisis. The nurse determines that the patient is very tense and might benefit from relaxation techniques. Which of the following is the best approach for the nurse to take when implementing this pain reduction technique? a) Ask parents and visitors to leave the room during this intervention. b) Close the door to the patient's room, dim the lights, and close the curtains before beginning. c) Begin the intervention with having the child breathe in and out quickly 10 times. d) Allow the television to remain on during this intervention to provide distraction for the patient.

Close the door to the patient's room, dim the lights, and close the curtains before beginning. Correct Explanation: Dimming the lights and closing the door to sounds, bright light, and distractions in the hall are good ways to begin a relaxation exercise. The television should be off during this technique so it will not be a distraction. Parents do not need to leave the room as this may cause increased anxiety for the child. Deep and slow breathing are relaxation techniques, not quick breathing

The nurse is caring for a child who has been sedated for a painful procedure. What is the priority nursing activity for this child? A. Place the child on a cardiac monitor B. Allow parents to stay with the child C. Monitor pulse oximetry D. Assess the child's respiratory effort

D. Assess the child's respiratory effort When the child is sedated for a procedure, it is very important for the nurse to actually visualize the child and his effort of breathing. Although equipment is important and is used routinely during sedation, it does not replace the need for visual assessment. Parents may be allowed to stay with the child, but assessment of breathing effort must take priority.

During shift report, the night nurse reports that the child who is terminally ill has developed tolerance to the morphine that the child has been receiving. The oncoming nurse realizes that the child: A. Is physically dependent on morphine B. Is addicted to morphine C. Is showing physical signs of withdrawal D. Will need more medication to achieve the same effect

D. Will need more medication to achieve the same effect Tolerance occurs when the body has become accustomed to the presence of the drug in the system. When this happens, the child will need more of a drug or a stronger drug to get the same effect. While the child may be physically dependent, this is not the meaning of tolerance. Addiction refers to a compulsive use of a substance despite harm. This is not the definition of tolerance. Withdrawal occurs when the opioid is stopped suddenly. This is not the meaning of tolerance.

The nurse is caring for a 6-year-old sickle-cell patient in an acute care setting. A high priority for this patient's plan of care is pain relief. The nurse understands that untreated acute pain can lead to which of the following physiological effects? a) Sleep disturbances, nocturnal enuresis, and impaired mobility b) Nausea, vomiting, migraine headaches, and developmental regression c) Constipation, nausea, and vomiting d) Impaired mobility, anorexia, anxiety, sleep disturbances, and developmental regression

Impaired mobility, anorexia, anxiety, sleep disturbances, and developmental regression Explanation: Unrelieved acute pain can lead to impaired mobility; anorexia, causing poor nutritional intake; delayed wound healing; anxiety and irritability; somatic symptoms; sleep disturbances; avoidance; developmental regression; and increased parental distress. Constipation, nausea, vomiting, nocturnal enuresis, and migraine headaches are not effects of acute pain

When administering parenteral or epidural opioids, the nurse should always have ready access to which medication? a) Antihistamines b) Anticonvulsants c) Benzodiazepines d) Naloxone e) Prostaglandin synthesis inhibitors

Naloxone Correct Explanation: Respiratory depression is a risk whenever parenteral or epidural opioids are used. Naloxone is a narcotic antagonist that is used to reverse the effects of respiratory depression. Antihistamines can reduce the itching side effect caused by some opioids. Prostaglandin synthesis inhibitors such as ibuprofen are sometimes used in conjunction with opioids to improve pain control; these do not depress respirations. Anticonvulsants can be effective for neuropathic pain. Benzodiazepines help to control anxiety, which is often associated with pain.

A. may reduce pain perception. A. Nonpharmacologic techniques for pain management may help the child with associated fears and stress related to pain. The strategies may provide assistance with coping that may reduce the perception of pain, decrease anxiety, and increase effectiveness of medications. B. The child with moderate or severe pain will require pharmacologic intervention. C. The child should be taught nonpharmacologic pain management strategies before pain occurs, thus reducing the implementation time. D. The child will still have the pain, but the perception may be altered.

Nonpharmacologic strategies for pain management: A. may reduce pain perception. B. make pharmacologic strategies unnecessary. C. usually take too long to implement. D. trick children into believing that they do not have pain.

The nurse realizes that many factors influence a child's pain experience. Which pain scale may be most appropriate to use with the African American 6-year-old? a) Oucher pain rating scale b) 0 to 10 Numeric pain intensity scale c) Pain Observation Scale for Young Children (POCIS) d) CRIES scale

Oucher pain rating scale Correct Explanation: The Oucher scale is a self-report scale appropriate for ages 3 to 13. Photographs of children's faces are used, indicating increasing degrees of distress from 0 to 10. There are three different scales for use with Caucasians, Hispanics, and African Americans. Both the age range and ethnicity suit this child. The 0 to 10 numeric scale can be used in children as young as 5 years and could be used with this child. The CRIES scale is meant to rate infant pain. The POCIS tool was developed for use with children ages 1 to 4 years.

The nurse is trying to assess the level of pain in a 3-year-old who appears to be experiencing abdominal pain. She lays out six photographs of children's faces representing "no hurt" to "biggest hurt you could ever have" and asks the child to point to the one that best shows what the child is feeling. Which pain scale is this nurse using? a) COMFORT behavior scale b) Wong-Baker FACES pain rating scale c) FLACC pain assessment tool d) Oucher pain rating scale

Oucher pain rating scale Explanation: The Oucher scale consists of six photographs of children's faces representing "no hurt" to "biggest hurt you could ever have." The Wong-Baker FACES pain rating scale consists of six cartoon-like faces ranging from smiling to tearful. The FLACC Pain Assessment Tool is a scale by which health care providers can rate a young child's pain when a child cannot give input, such as during circumcision. It incorporates five types of behaviors that can be used to rate pain: facial expression, leg movement, activity, cry, and consolability. The COMFORT behavior scale is a pain rating scale devised by nurses to rate pain in very young infants. On the first part of the scale, six different categories (alertness, calmness/agitation, crying, physical movement, muscle tone, and facial expression) are rated from 1 to 5.

The nurse is planning immediate postoperative care for an infant after a cleft-lip repair. Which of the following should the plan include? a) Allow the infant to be as active as possible after surgery. b) Crying is good for the infant to decrease risk of pneumonia after anesthetic. c) Pain medication should be given on a routine basis. d) Encourage use of pacifier after surgery.

Pain medication should be given on a routine basis. Correct Explanation: After any surgery on a child, the plan should include pain medication administration on a routine basis. The infant will not be able to request pain medication and will need to rest the surgical site. Providing pain medication will help the infant in the postoperative period.

The nurse is preparing a toddler for the repair of an eyebrow laceration. The girl is most likely to demonstrate which of the following responses in anticipation of the procedure? a) Remain outwardly calm and ask numerous questions. b) Scream and cling tightly to her parent. c) Stare out the window while clenching the hands. d) Attempt to postpone the procedure by asking to "go potty."

Scream and cling tightly to her parent. Correct Explanation: A toddler is most likely to show regressive behaviors such as clinging and crying loudly. Preschoolers may say they need to go to the bathroom or get an item from another place to try to postpone the procedure. School-age children are more likely to withdraw into supposed inattention or silence and show muscular tension. Adolescents may look stoic in order to appear in control of themselves, or they may ask many questions (intellectualizing).

The nurse working in the emergency department is caring for an 8-year-old male who was hit by a car while running across the street and has suffered extensive abrasions, contusions, and broken bones. IV morphine analgesia has been given. The child is unable to follow simple directions to allow for procedures to be completed. Which of the following is the best action to take for this child? a) Explain the procedure to the child. b) Give the child another dose of morphine. c) The child should be sedated. d) Restrain the child in order to keep him safe.

The child should be sedated. Correct Explanation: If a child is unable to follow directions and allow a procedure to proceed safely with analgesia alone, the child should be sedated. Children are routinely restrained without sedation or analgesia for painful procedures, something that would be unthinkable with an adult. This would not be an appropriate intervention. The child in is too much pain to understand an explanation at this point.

The nurse is caring for a term infant suffering from meconium aspiration in the nursery. The nurse reviews orders for a peripherally inserted central catheter (PICC) line placement and intubation. Which of the following statements demonstrates the nurse's knowledge of painful procedures and the newborn? a) The newborn does not have fully developed pain receptors, and therefore needs little or no pain medication. b) The newborn will not remember pain and does not need analgesia for painful procedures. c) Newborns are rarely subjected to painful procedures without anesthesia. d) The newborn's pain pathway components are developed enough at birth to experience pain.

The newborn's pain pathway components are developed enough at birth to experience pain. Correct Explanation: Neuroanatomical and neuroendocrine components of the pain pathway are sufficiently developed in the neonate to allow the transmission and perception of pain. While infants may not remember painful experiences as distinct actual events, the functional structures for long-term memory, specifically, the integrity of the limbic system and diencephalons, are well developed in newborns. These early painful experiences may be stored as procedural memory, not accessible to conscious recall. Ample evidence indicates that both term and preterm neonates have the capacity to experience and remember pain much like older children and adults do. Newborns should receive analgesia for painful procedures.

When assessing a wound for proper anesthetic effect, which of the following findings would indicate the wound would be ready for suturing? a) The nurse can visualize a blanching effect in the wound bed. b) The nurse can visualize a blue tone to the wound bed. c) The nurse can visualize fresh bleeding in the wound bed. d) The nurse can visualize a reddened appearance of the wound bed.

The nurse can visualize a blanching effect in the wound bed. Correct Explanation: When assessing readiness of an anesthetic agent, look for blanching of the wound bed to assess effectiveness. Redness, a blue tone, and fresh bleeding would not indicate a wound is properly anesthetized.

C. plan a preventive schedule of pain medication around the clock. A. This is appropriate for the immediate pain but will not facilitate the more long-term plan of pain management. B. The dosage of analgesic is increased until pain is controlled, not until sedation is adequate. C. An around-the-clock administration strategy should be used for a child recovering from trauma and surgery. This schedule will help prevent low plasma levels of the drug, leading to breakthrough pain. D. The child should be frequently assessed for pain, and medication doses titrated accordingly. It is inappropriate to give a child a clock with instructions as to when pain medication can be given, especially a child who has experienced a traumatic event.

The nurse is caring postoperatively for an 8-year-old child with multiple fractures and other trauma resulting from a motor vehicle injury. The child is experiencing severe pain. An important consideration in managing the child's pain is to: A. give only an opioid analgesic at this time. B. increase the dosage of analgesic until the child is adequately sedated. C. plan a preventive schedule of pain medication around the clock. D. give the child a clock and explain when he or she can have pain medications.

D. children often demonstrate increased behavioral signs of discomfort with repeated painful procedures. A. There are no data to support the theory that children tolerate pain better than adults. B. The child has increasing difficulty with numerous and repeated painful procedures rather than becoming accustomed to them. C. Pain is whatever the experiencing person defines it to be. D. Children with chronic illnesses are more likely to identify invasive procedures as stressful compared with children with acute illnesses.

The nurse is starting an intravenous (IV) line on a school-age child with cancer. The child says, "I have had a million IVs. They hurt." The nurse's response should be based on the knowledge that: A. children tolerate pain better than adults. B. children become accustomed to painful procedures. C. children often lie about experiencing pain. D. children often demonstrate increased behavioral signs of discomfort with repeated painful procedures.

A 12-year-old girl needs a lumbar puncture to collect cerebral spinal fluid for laboratory exam plus injection of medication into the central nervous system. She expresses great fear of the procedure because of anticipated pain and the inability to hold still. The nurse contacts the physician to make which suggestion? a) The use of conscious sedation for the lumbar puncture. b) Delay the procedure until the child can achieve better understanding and acceptance. c) Administration of an oral antianxiety medication prior to the procedure. d) Include the child's parents and a child life specialist in the procedure room.

The use of conscious sedation for the lumbar puncture. Explanation: The nurse recognizes the child's fear and is acting as her advocate suggesting the use of conscious sedation. It will be the most effective way to relieve the child's anxiety, pain, and concern about cooperation. A medication given for anxiety prior to the procedure may ameliorate some stress and make lying still a bit easier but will not relieve pain. Support from parents and a child life specialist is helpful and can be part of the conscious sedation plan. Alone it would not be adequate to assist the child. Delaying the procedure to do additional teaching could be helpful in some situations but not the best choice here.

The nurse is caring for an adolescent postoperative patient in an acute care setting with PRN pain medication orders of Tylenol and Vicodin. Which of the following should the nurse closely monitor? a) Medication interactions between these drugs b) Side effects such as diarrhea, dry mouth, and irritability c) Total daily dosage of acetaminophen due to liver toxicity d) Physical dependency on these medications

Total daily dosage of acetaminophen due to liver toxicity Correct Explanation: Both Tylenol and Vicodin contain acetaminophen. Total daily doses of acetaminophen should be monitored closely due to the toxic effects it can have on the liver. These drugs would not have any adverse interactions with each other. Physical dependency is not a concern when using opioid medications for a short period of time, such as a postoperative period. Side effects of opioids are constipation, sedation, and respiratory depression.

B. administer morphine sulfate immediate release (MSIR) intravenously (IV). A. Intramuscular injections should be avoided in cancer patients because of increased risk of bleeding and the fact that they do not act immediately. B. The nurse should administer an immediate-release opioid such as MSIR IV for the breakthrough pain. C. Nonpharmacologic strategies are not effective in severe pain. D. Transdermal fentanyl will take up to 24 hours to reach peak effect and thus is not effective for severe breakthrough pain.

Transdermal fentanyl (Duragesic) is being used for an adolescent with cancer who is in hospice care. The adolescent has been comfortable for several hours but now complains of severe pain. The most appropriate nursing action is to: A. administer meperidine (Demerol) intramuscularly (IM). B. administer morphine sulfate immediate release (MSIR) intravenously (IV). C. use a nonpharmacologic strategy. D. place another fentanyl patch on the adolescent.

The nurse is caring for a 2-year-old postoperative PET patient. Which of the following considerations is the most appropriate for this patient's developmental stage? a) Fears bodily mutation or injury b) Uses words for pain such as owie, boo-boo, or hurt c) Uses delays to put off treatment d) Understands time

Uses words for pain such as owie, boo-boo, or hurt Correct Explanation: The toddler uses simple terms to describe pain, such as owie, boo-boo, or hurt. School-aged and preschool-aged children fear bodily mutilation. Preschool-aged children delay or put off treatment and school-aged children understand time.

The nurse is assisting with the administration of parenteral opioids for a child for an initial dose. What action should the nurse take first? a. ensure naloxone is readily available b. assess for any adverse reaction c. assess the status of bowel sounds d. premedicate with acetaminophen

a. ensure naloxone is readily available

A nurse is caring for a 4 year old child who is exhibiting extreme anxiety and behavior upset prior to receiving stitches for a deep chin laceration. Which of the following is the priority nursing intervention? a. Ensuring that emergency equipment is readily available b. serving as an advocate for the family to ensure appropriate pharmacologic agents are chosen c. conducting an initial assessment of pain to serve as a baseline from which options for relief can be chosen d. ensuring the lighting is adequate for the procedure but not so bright to cause discomfort

b. serving as an advocate for the family to ensure appropriate pharmacologic agents are chosen

The nurse is providing teaching the parents of a 9 year old boy with episodes of chronic pain how to help manage his pain nonpharmacologically. Which of the following statements indicates a need for further teaching? a. We should perform the techniques along with him b. we should start the method as soon as he feels pain c. we need to identify the ways in which he shows pain d. we should select a method that he likes the best

b. we should start the method as soon as he feels pain

The nurse is assigned to care for a 14 year old child who is hospitalized in traction for serious leg fractures after an automobile accident. The parents ask the nurse to avoid administering analgesics to their child to help prevent him from becoming addicted. What response by the nurse is indicated? a. We can talk with the physician to see about reducing the amount of medications given to reduce the potential for addiction b. if there is no history of drug abuse in the family there should be no increased risk for the development of addiction d. your child is too young to experience drug addiction

c

The nurse is caring for a 5 year old child who underwent a painful surgical procedure earlier in the day. The nurse notes the child has not reported pain to any of the nursing staff. What action by the nurse is indicated? a. contact the physician to report the child's condition b. administer prophylactic analgesics c. observe for behavioral cues consistent with pain d. encourage the child to report pain

c.

The nurse is preparing to administer a dose of Toradol (ketorolac) to a 15 year old child. The nurse should do which of the following to reduce the potential for gastrointestinal upset? a. administer the medication before meals b. administer the medication with milk c. administer the medication with meals d. administer the medication with a citrus beverage

c.

A nurse is applying EMLA as ordered. The nurse understands that EMLA is contraindicated in which of the following situations? a. infants less than 6 weeks of age b. children with darker skin c. infants less than 12 months receiving methemoglobin-inducing agents. d children undergoing venous cannulation or intramuscular injections

c. infants less than 12 months receiving methhemoglobin-inducing agents

An injured child has been hospitalized. The physician orders an application to help constrict capillaries and therefore reduce capillary permeability and edema. Which does the nurse apply to the patient? a) cold pack b) heat

cold pack Correct Explanation: Cold reduces pain by constricting the capillaries and therefore reducing vessel permeability, edema, and pressure at an injured site.

A 4-year-old patient is scheduled for an MRI. The child's mother is informed that the child will be free of pain but sedated to ensure stillness during the procedure. Which type of anesthesia does the nurse expect this child to have? a) general anesthesia b) conscious sedation c) IM injection d) PCA

conscious sedation Correct Explanation: Conscious sedation refers to a state of depressed consciousness, usually obtained through IV analgesia therapy. The technique allows a child to be both pain free and sedated for a procedure.

A pregnant teen voices concerns related to potential paralysis about the plans for an epidural anesthetic to be administered. What information can be provided to the teen? a. paralysis is not a serious concern for the procedure b. the spinal cord will not be damaged by the insertion of the epidural catheter c. the spinal cord ends above the area where the epidural is inserted d. the risk of paralysis is limited because your physician is skilled in the administration of epidurals

d.

The nurse is caring for a patient who is 30 weeks gestation. .The patient is preparing to undergo an invasive procedure on her unborn baby. The patient discusses the likelihood that her fetus will experience pain. Which of the following statements indicates an understanding of the influences of stimuli on the unborn fetus? a. unborn babies do not feel painful sensations b. since my child is a boy the amount of pain that he can experience is lessened c. painful stimuli can be felt by the fetus only in the hours prior to delivery d. the physiological maturity needed for the fetus to sense pain is present by about 23 weeks gestation

d.

A nurse is preparing to give an injection to a 7 year old who appears afraid and hesitant. The nurse tells the patient that it is OK to say "ouch" when the shot is given. This is an example of which of the following pain control techniques? a) distraction b) anxiety reduction c) cutaneous stimulation d) guided imagery

distraction Correct Explanation: Telling a child to say "ouch" while an injection is administered is the simplest use of distraction.

You teach a child to imagine that she is swimming in a cool, shady park where nothing can harm her during a blood-drawing procedure. This technique is called a) imagery. b) thought stopping. c) nerve stimulation. d) park therapy.

imagery. Correct Explanation: Because children have keen imaginations, they are able to substitute a pleasant thought for an unpleasant one (substitution of meaning or imagery).

A young child is hospitalized with pneumonia. Upon admission he informs the nurse that he is not having pain but just a bad cough. A few hours later, the child he begins to complain of pain in his right lower back. This first report of feeling pain refers to which of the following? a) pain peak b) pain duration c) pain tolerance d) pain threshold

pain threshold Correct Explanation: Pain threshold refers to the point at which the child first feels the pain.

You teach a child to use a FACES pain rating scale prior to surgery. At that time, she points to the smiling face. Following surgery when you suspect she has pain, she points again to the smiling face. You would interpret this as a) you must be interpreting her degree of pain falsely. b) she is using the scale to predict what she would like, not what she has. c) she has difficulty focusing on the right side of the scale. d) she does not have pain.

she is using the scale to predict what she would like, not what she has. Correct Explanation: Preschoolers use "magical thinking," or believe that what they wish will come true. They may use pain scales, therefore, to "wish" for a smiling face, rather than for rating their pain.

A young boy is in the emergency department with swelling and pain in the right ankle. He states that he was playing soccer, somehow twisted the ankle, and could not walk off the field. The physician tells the patient that it is only a bad sprain. Which type of pain is this patient experiencing? a) visceral b) somatic c) cutaneous d) chronic

somatic Correct Explanation: Somatic pain originates from deep body structures, such as muscles or blood vessels. The pain of a sprained ankle is somatic pain.

A 5 year old arrives at the emergency department and reports abdominal pain. After performing an assessment and laboratory work, the physician diagnoses appendicitis. The nurse knows that this child is experiencing which type of pain? a) somatic b) cutaneous c) chronic d) visceral

visceral Correct Explanation: Visceral pain involves sensations that arise from internal organs, such as the intestines. The pain of appendicitis is visceral pain.

The nurse is assessing the abdomen of a preschooler admitted for lower right quadrant pain. Which questions or requests are appropriate to use with this child? Select all that apply. a) "Is your hurt getting better?" b) "Does your tummy ache?" c) "Touch the spot on your tummy where it hurts." d) "The hurt—is it sharp or dull?" e) "Choose the face that shows how you feel now." (FACES pain rating scale)

• "Touch the spot on your tummy where it hurts." • "Choose the face that shows how you feel now." (FACES pain rating scale) Correct Explanation: A preschooler is able to tell someone or indicate where it hurts. Starting at age 3 years, most can effectively use the FACES scale. However, preschoolers will have difficulty distinguishing between types and intensity of pain and even if pain is better or worse.

A child with Down syndrome has had surgery and experiences periodic pain. The child is 13 year olds but functions much like an 8-year-old. Which pain scales may be appropriate to use with this girl? Select all that apply. a) Adolescent Pediatric Pain Tool b) FACES pain rating scale c) Poker chip tool d) Word-graphic rating scale

• FACES pain rating scale • Poker chip tool Correct Explanation: The poker chip tool has been successfully used for those 4 years of age and older. The child stacks pieces of hurt. This is concrete and would be a potential choice for use with this girl. The FACES scale can be used in children as young as 3 years. The faces are generic line drawings indicating increasing degrees of distress. The child points to the one indicating how he or she feels. This could be effective with the girl who functions as an 8-year-old. The other scales are not likely to work as well for rating this child's pain. The Adolescent Pediatric Pain Tool is useful with children ages 8 years to age 17 years. The hurt is colored, a scale rates severity, and a word list is used to describe pain. This is most likely too complex for the child with Down syndrome. The same is true for the word-graphic scale. Children between 4 years and 17 years have used it. The child is asked to indicate the level of pain on the scale following an explanation of the descriptors. This activity may not hold the attention of the child with Down syndrome and may be too wordy for adequate comprehension. Often children regress to an earlier developmental level when stressed by illness, and a simpler scale is more effective. When possible, teaching the child to use the scale prior to a painful experience aids in getting reliable feedback.

The adolescent receiving morphine IV for pain control needs which of the following included in his nursing care plan (NCP)? Select all that apply. a) Monitoring for itching b) High-fat diet c) Assessment for suppressed cough reflex d) Stand-by assistance when using the bathroom e) Naloxone readily available

• Monitoring for itching • Stand-by assistance when using the bathroom • Naloxone readily available Explanation: Naloxone should be readily available to reverse possible respiratory depression, a side effect of morphine. Dizziness and sedation are likely to accompany this pain control. Therefore, safety measures such as assistance when getting up to go to the bathroom or ambulating plus use of side rails are important inclusions in the NCP. Itching, particularly of the face and hands, is a relatively common side effect of morphine sulfate. It can be treated with cool compresses or an antihistamine and included in the plan of care. A high-fat diet and assessing for a suppressed cough reflex are interventions not needed for the portion of this adolescent's NCP that addresses pain control through use of IV morphine.

The nurse wishes to reassure a 3-year-old girl that there will be no discomfort when her peripheral IV site is examined. The nurse will use which words? Select all that apply. a) No owie b) No pain c) No boo-boo d) No hurt

• No hurt • No owie • No boo-boo Explanation: Children at 2½ years will spontaneously use the word "hurt." Other common words for pain used by toddlers or young preschoolers are "owie" and "boo-boo." Children tend not to use the word "pain" until around 6 years of age. Individual children will have their unique descriptors for pain. Nurses need to know these and use them when assessing for pain.

An aunt at the bedside of a 7-year-old holds the child's hand and gently traces her fingers up and down the child's arm while talking softly about pleasant experiences on the grandparents' farm. This relative is using what technique to reduce pain? Select all that apply. a) Nonpharmacologic management b) Relaxation c) Behavioral-cognitive strategy d) Biofeedback e) Positive self-talk

• Nonpharmacologic management • Behavioral-cognitive strategy • Relaxation Explanation: The aunt is promoting relaxation through hand holding, stroking, and soothing talk. The soft talk of pleasant experiences is also a cognitive strategy that helps the child focus on something other than pain. Both belong to the broader category of nonpharmacologic management since medication use is not involved. Biofeedback teaches a child to modify certain body functions. It is taught by a specialist in advance of the anticipated pain. Positive self-talk uses positive statements the child makes to himself or herself to change the focus of the pain experience. Neither biofeedback nor positive self-talk are in use here.

A nursing student correctly identifies that heat is primarily used after the first 24 hours of an injury and does which of the following? (Select all that apply.) a) Reduces edema b) Increases blood flow to the area c) Reduces vessel permeability d) Dilates capillaries e) Constricts capillaries

• Reduces edema • Increases blood flow to the area • Dilates capillaries Correct Explanation: After the first 24 hours of an injury, heat dilates capillaries, increases blood flow to the area, and helps reduce edema.

What does the nursing instructor list for students as the major steps of pain conduction? (Select all that apply.) a) perception b) transduction c) transformation d) evaporation e) transmission f) modulation

• transduction • transmission • perception • modulation Explanation: The four major steps of pain conduction are transduction, transmission, perception, and modulation.


Kaugnay na mga set ng pag-aaral

AUTOMATIC EMERGENCY BRAKING AND INTELLIGENT FORWARD COLLISION WARNING

View Set

THE NATURE AND PROCESS OF COMMUNICATION

View Set

Campbell Biology 8th Ed Test Bank chapter 24, 26-31

View Set